2006 AMC 8 Problems/Problem 1

Revision as of 20:14, 5 September 2011 by Math Kirby (talk | contribs) (Created page with "== Problem == Mindy made three purchases for <math> \textdollar 1.98</math> dollars, <math> \textdollar 5.04 </math> dollars, and <math> \textdollar 9.89</math> dollars. What was...")
(diff) ← Older revision | Latest revision (diff) | Newer revision → (diff)

Problem

Mindy made three purchases for $\textdollar 1.98$ dollars, $\textdollar 5.04$ dollars, and $\textdollar 9.89$ dollars. What was her total, to the nearest dollar?

$\textbf{(A)}\ 10\qquad\textbf{(B)}\ 15\qquad\textbf{(C)}\ 16\qquad\textbf{(D)}\ 17\qquad\textbf{(E)}\ 18$

Solution

The three prices round to $\textdollar 2$, $\textdollar 5$, and $\textdollar 10$, which has a sum of $\boxed{\textbf{(D)}\ 17}$